K
Khách

Hãy nhập câu hỏi của bạn vào đây, nếu là tài khoản VIP, bạn sẽ được ưu tiên trả lời.

10 tháng 1 2022

câu 2: 

Với p=2→2p+1=5p=2→2p+1=5 không là lập phương 11 số tự nhiên

→p=2→p=2 loại

→p>2→(p,2)=1→p>2→(p,2)=1

Đặt 2p+1=(2k+1)3,k∈N2p+1=(2k+1)3,k∈N vì 2p+12p+1 lẻ

→2p=(2k+1)3−1→2p=(2k+1)3−1

→2p=(2k+1−1)((2k+1)2+(2k+1)+1)→2p=(2k+1−1)((2k+1)2+(2k+1)+1)

→2p=2k(4k2+6k+3)→2p=2k(4k2+6k+3)

→p=k(4k2+6k+3)→p=k(4k2+6k+3)

Vì pp là số nguyên tố, 4k2+6k+3>k4k2+6k+3>k

→k=1→k=1 và 4k2+6k+34k2+6k+3 là số nguyên tố

→4k2+6k+3=13→4k2+6k+3=13 (Khi k=1k=1) là số nguyên tố

→k=1→k=1 chọn

→2p+1=27→2p+1=27

→p=13

câu 3: p−q+2q=(p−q)3→2q=(p−q)((p−q)2−1)=(p−q)(p−q−1)(p−q+1)p−q+2q=(p−q)3→2q=(p−q)((p−q)2−1)=(p−q)(p−q−1)(p−q+1)
Th1: p−qp−q chia hết cho 2 suy ra p−q=2kp−q=2k
Suy ra q=k.(2k−1)(2k+1)q=k.(2k−1)(2k+1)
Do vậy k=1k=1 vì nếu không thì qq thành tích 3 số nguyên lớn hơn 1 suy ra vô lý vì nó là nguyên tố.
Suy ra p−q=2p−q=2 Như vậy q=3,p=5q=3,p=5 Thỏa mãn
TH2: p−q−1p−q−1 chia hết cho 2 suy ra p−q−1=2tp−q−1=2t nên q=(2t+1)t(2t+2)q=(2t+1)t(2t+2)
Do vậy t=0t=0 vì nếu không thì qq thành tích 2 số nguyên lớn hơn 1.
Suy ra p−q−1=0↔p−q=1↔p=3,q=2p−q−1=0↔p−q=1↔p=3,q=2 thay vào đề loại.
TH3: p−q+1=2mp−q+1=2m suy ra q=(2m−1)(2m−2)mq=(2m−1)(2m−2)m
Nếu m≥2m≥2 suy ra qq thành tích 3 số nguyên lớn hơn 1 loại
Suy ra m=0,1m=0,1 thay vào đều loại.
Vậy p=5,q=3p=5,q=3

tick nha

Nhìn là cũng biết e cop rùi :))

Khi cop nếu ko chú ý thì sẽ bị ra mỗi cái hai lần, mà e cũng thế.

=> Chứng tỏ cop. Quá chuẩn nhỉ?

NV
12 tháng 1 2022

1.

\(\left(a+b+c\right)^3=a^3+b^3+c^3+3\left(a+b+c\right)\left(ab+bc+ca\right)-3abc\)

Do vế phải chia hết cho 3  \(\Rightarrow\) vế trái chia hết cho 3

\(\Rightarrow a+b+c⋮3\Rightarrow\left(a+b+c\right)^3⋮27\)

\(a+b+c⋮3\Rightarrow3\left(a+b+c\right)⋮9\)

\(\Rightarrow\left(a+b+c\right)^3-\left(a^3+b^3+c^3\right)-3\left(a+b+c\right)\left(ab+bc+ca\right)⋮9\)

\(\Rightarrow3abc⋮9\Rightarrow abc⋮3\)

2.

Đặt \(2p+1=n^3\Rightarrow2p=n^3-1=\left(n-1\right)\left(n^2+n+1\right)\) (hiển nhiên n>1)

Do \(n^2+n+1=n\left(n+1\right)+1\) luôn lẻ \(\Rightarrow n-1\) chẵn \(\Rightarrow n=2k+1\)

\(\Rightarrow2p=\left(2k+1-1\right)\left(n^2+n+1\right)=2k\left(n^2+n+1\right)\)

\(\Rightarrow p=k\left(n^2+n+1\right)\Rightarrow k=1\Rightarrow n=3\)

\(\Rightarrow p=13\)

12 tháng 1 2022

Tham khảo:

2, Với \(p=2->2p+1=5\) không là lập phương 1 số tự nhiên

\(->p=2\) loại

\(-> p>2->(p,2)=1\)

Đặt \(2p+1=(2k+1)^3, k∈ N,\)vì \(2p+1\) lẻ

\(->2p=(2k+1)^3-1\)

\(-> 2p=(2k+1-1)[(2k+1)^2+(2k+1)+1]\)

\(->2p=2k(4k^2+6k+3)\)

\(->p=k(4k^2+6k+3)\)

Vì \(p\)  là số nguyên tố, \(4k^2+6k+3>k\)

\(->k=1\) và \(4k^2+6k+3\) là số nguyên tố.

\(->4k^2+6k+3=13(\) khi \(k=1)\) là số nguyên tố

\(->k=1\) (chọn)

\(-> 2p+1=27\)

\(->p=13\)

27 tháng 5 2017

theo cong thuc  x1 x2

2,Giải: 

♣ Ta thấy p = 2 thì 2p + 1 = 5 không thỏa = n³ 

♣ Nếu p > 2 => p lẻ (Do Số nguyên tố chẵn duy nhất là 2 ) 
Mặt khác : 2p + 1 là 1 số lẻ => n³ là một số lẻ => n là một số lẻ 

=> 2p + 1 = (2k + 1)³ ( với n = 2k + 1 ) 
<=> 2p + 1 = 8k³ + 12k² + 6k + 1 
<=> p = k(4k² + 6k + 3) 

=> p chia hết cho k 
=> k là ước số của số nguyên tố p. 

Do p là số nguyên tố nên k = 1 hoặc k = p 

♫ Khi k = 1 
=> p = (4.1² + 6.1 + 3) = 13 (nhận) 

♫ Khi k = p 
=> (4k² + 6k + 3) = (4p² + 6p + 3) = 1 
Do p > 2 => (4p² + 6p + 3) > 2 > 1 
=> không có giá trị p nào thỏa. 

Đáp số : p = 13

1 tháng 11 2018

tai sao b^c +a +a^b +c +c^a+b=2(a+b+c)

1. Giả sử p và q là các số nguyên sao cho: \(\frac{p}{q}=1-\frac{1}{2}+\frac{1}{3}-\frac{1}{4}+.....-\frac{1}{1334}+\frac{1}{1335}\)CMR: \(P⋮2003\)2. CM:\(\forall n\in N,n\ge2\)thì\(An=2^{2^n}+4⋮10\)3.CM: \(\forall n\in N,n\ge1\)thì \(Bn=4^n+15n-1⋮9\)4.CM: \(\forall n\in Z,n\ge0\)thì \(Cn=2^{3^n}+1⋮3n+1\)nhưng \(⋮̸3^n+2\)5.CM:tổng hợp phương của 3 số tự nhiên liên tiếp n,n+1,n+2\(⋮9\forall n\ge0\)6. Cm: A=\(\frac{5^{125}-1}{5^{25}-1}\)không phải là...
Đọc tiếp

1. Giả sử p và q là các số nguyên sao cho: \(\frac{p}{q}=1-\frac{1}{2}+\frac{1}{3}-\frac{1}{4}+.....-\frac{1}{1334}+\frac{1}{1335}\)

CMR: \(P⋮2003\)

2. CM:\(\forall n\in N,n\ge2\)thì\(An=2^{2^n}+4⋮10\)

3.CM: \(\forall n\in N,n\ge1\)thì \(Bn=4^n+15n-1⋮9\)

4.CM: \(\forall n\in Z,n\ge0\)thì \(Cn=2^{3^n}+1⋮3n+1\)nhưng \(⋮̸3^n+2\)

5.CM:tổng hợp phương của 3 số tự nhiên liên tiếp n,n+1,n+2\(⋮9\forall n\ge0\)

6. Cm: A=\(\frac{5^{125}-1}{5^{25}-1}\)không phải là một số nguyên tố 

7.Tìm tất cả các số nguyên tố P sao cho tổng của tất cả các ước số tự nhiên của các phương trình là 1 số chính phương

8. Biết P và \(8p^2-1\)cũng là số nguyên tố

9. Tìm tất cả các số nguyên tố có 4 chữ số \(\overline{abcd}\)sao cho \(\overline{ab}\)\(\overline{ac}\)là các số nguyên tố và \(b^2=\overline{cd}+b-c\)

10.Cho \(\overline{abc}\)là 1 số nguyên tố. CM phương trình: \(ax^2+bx+c=0\)không có nghiệm hữu tỉ

 

0
10 tháng 12 2018

Từ gt \(\Rightarrow ab-ac-bc+c^2=c^2\)

        \(\Leftrightarrow ab=ac+bc\)

       \(\Leftrightarrow ab=c\left(a+b\right)\)

       \(\Leftrightarrow abc=c^2\left(a+b\right)\)

Bây giờ chỉ cần chứng minh ( a + b ) là số chính phương nx là xog !

Gọi \(ƯCLN\left(a-c;b-c\right)=d\left(d\inℕ^∗\right)\)

\(\Rightarrow\hept{\begin{cases}a-c⋮d\\b-c⋮d\end{cases}\Rightarrow}\left(a-c\right)-\left(b-c\right)⋮d\)

                            \(\Rightarrow a-b⋮d\)

Mà \(\left(a;b\right)=1\)

\(\Rightarrow d=1\)

Hay \(\left(a-c;b-c\right)=1\)

Mà \(\left(a-c\right)\left(b-c\right)=c^2\)là số chính phường

Nên a - c và b - c đều là số chính phương

Đặt \(\hept{\begin{cases}a-c=x^2\\b-c=y^2\end{cases}\left(x;y\inℕ\right)}\)

\(\Rightarrow x^2.y^2=\left(a-c\right)\left(b-c\right)\)

\(\Leftrightarrow x^2y^2=c^2\)

\(\Leftrightarrow xy=c\)( Do xy và c đều dương )

Ta có : \(\left(a-c\right)+\left(b-c\right)=x^2+y^2\)

\(\Leftrightarrow a+b-2c=x^2+y^2\)

\(\Leftrightarrow a+b=x^2+2c+y^2\)

\(\Leftrightarrow a+b=x^2+2xy+y^2\)

\(\Leftrightarrow a+b=\left(x+y\right)^2\)là số chính phương

Do đó : \(abc=c^2.\left(x+y\right)^2=\left(cx+cy\right)^2\)là số chính phương

Vậy .................